Đến nội dung

tuaneee111 nội dung

Có 170 mục bởi tuaneee111 (Tìm giới hạn từ 16-05-2020)



Sắp theo                Sắp xếp  

#683925 $ \frac{1}{a+b+1}+\frac{1}{...

Đã gửi bởi tuaneee111 on 10-06-2017 - 16:13 trong Bất đẳng thức và cực trị

Cho ba số dương a,b,c thỏa mãn abc=1. Chứng minh rằng:

$ \frac{1}{a+b+1}+\frac{1}{b+c+1}+\frac{1}{c+a+1} \leq 1 $

Bài này quá quen thuộc rồi, mình xin đóng góp thêm một cách nữa.

Đặt $ \displaystyle \left( {a,b,c} \right)\to \left( {{{x}^{3}},{{y}^{3}},{{z}^{3}}} \right)$. Bất đẳng thức cần chứng minh tương đương: $$\frac{1}{{{x^3} + {y^3} + xyz}} + \frac{1}{{{y^3} + {z^3} + xyz}} + \frac{1}{{{z^3} + {x^3} + xyz}} \leqslant 1$$Ta có bổ đề sau: ${a^3} + {b^3} \geqslant ab\left( {a + b} \right)$. Áp dụng ta được: $$\sum\limits_{cyc} {\frac{1}{{{x^3} + {y^3} + xyz}}}  \leqslant \sum\limits_{cyc} {\frac{1}{{xy\left( {x + y} \right) + xyz}}}  = \sum\limits_{cyc} {\frac{1}{{xy\left( {x + y + z} \right)}}}  = 1 \Rightarrow Q.E.D$$




#683924 $$3\left( {{a^3} + {b^3} + {c^3}} \right) + 2abc \ge...

Đã gửi bởi tuaneee111 on 10-06-2017 - 16:09 trong Bất đẳng thức và cực trị

P/s: Lời giải tương đối cơ bắp nha :D ...

Đặt $f\left( {a,b,c} \right) = 3\left( {{a^3} + {b^3} + {c^2}} \right) + 2abc - 11\sqrt {{{\left( {\frac{{{a^2} + {b^2} + {c^2}}}{3}} \right)}^3}} $. Do $f\left( {a,b,c} \right)$ là hàm thuần nhất nên không mất tính tổng quát ta giả sử $ \displaystyle \left\{ \begin{array}{l}{{a}^{2}}+{{b}^{2}}+{{c}^{2}}=3\\a=\text{min}\left\{ {a,b,c} \right\}\end{array} \right.$. Khi đó cần chứng minh $f\left( {a,b,c} \right) = 3\left( {{a^3} + {b^3} + {c^3}} \right) + 2abc - 11 \geqslant 0$. Ta có: $$f\left( {a.b.c} \right) - f\left( {a,\sqrt {\frac{{{b^2} + {c^2}}}{2}} ,\sqrt {\frac{{{b^2} + {c^2}}}{2}} } \right) = \frac{1}{2}\left[ {3\left( {2\left( {{b^3} + {c^3}} \right) - \left( {{b^2} + {c^2}} \right)\sqrt {2\left( {{b^2} + {c^2}} \right)} } \right) - a{{\left( {b - c} \right)}^2}} \right]$$$$ = \frac{1}{2}\left[ {3\left( {\sqrt {2\left( {{b^2} + {c^2}} \right)}  - b - c} \right)\left( {\left( {b + c} \right)\sqrt {2\left( {{b^2} + {c^2}} \right)}  + 2bc} \right) - a{{\left( {b - c} \right)}^2}} \right]$$$$ = \frac{1}{2}.{\left( {b - c} \right)^2}\left[ {\frac{{3\left( {\left( {b + c} \right)\sqrt {2\left( {{b^2} + {c^2}} \right)}  + 2bc} \right)}}{{\sqrt {2\left( {{b^2} + {c^2}} \right)}  + b + c}} - a} \right]$$Do $ \displaystyle a=\min \left\{ {a,b,c} \right\}\Rightarrow \left\{ \begin{array}{l}a\in \left[ {0;1} \right]\\b+c\ge 2\end{array} \right.$. Khi đó cần chứng minh: $\frac{{3\left( {\left( {b + c} \right)\sqrt {2\left( {{b^2} + {c^2}} \right)}  + 2bc} \right)}}{{\sqrt {2\left( {{b^2} + {c^2}} \right)}  + b + c}} - a > 0$Ta có: $$\frac{{3\left( {\left( {b + c} \right)\sqrt {2\left( {{b^2} + {c^2}} \right)}  + 2bc} \right)}}{{\sqrt {2\left( {{b^2} + {c^2}} \right)}  + b + c}} - a \geqslant \frac{{3\left( {\left( {b + c} \right)\sqrt {2\left( {{b^2} + {c^2}} \right)}  + 2bc} \right)}}{{\sqrt {2\left( {{b^2} + {c^2}} \right)}  + b + c}} - 1$$$$ = \frac{{\left( {3b + 3c - 1} \right)\sqrt {2\left( {{b^2} + {c^2}} \right)}  + 6bc - b - c}}{{\sqrt {2\left( {{b^2} + {c^2}} \right)}  + b + c}} \geqslant \frac{{5\sqrt {2\left( {{b^2} + {c^2}} \right)}  - b - c + 6bc}}{{\sqrt {2\left( {{b^2} + {c^2}} \right)}  + b + c}}$$$$ > \frac{{\sqrt {2\left( {{b^2} + {c^2}} \right)}  - b - c + 6bc}}{{\sqrt {2\left( {{b^2} + {c^2}} \right)}  + b + c}} = \frac{{\frac{{{{\left( {b - c} \right)}^2}}}{{\sqrt {2\left( {{b^2} + {c^2}} \right)}  + b + c}} + 6bc}}{{\sqrt {2\left( {{b^2} + {c^2}} \right)}  + b + c}} \geqslant 0$$Vậy ta được: $f\left( {a,b,c} \right) \geqslant f\left( {a,\sqrt {\frac{{{b^2} + {c^2}}}{2}} ,\sqrt {\frac{{{b^2} + {c^2}}}{2}} } \right) = f\left( {a,\sqrt {\frac{{3 - {a^2}}}{2}} ,\sqrt {\frac{{3 - {a^2}}}{2}} } \right)$

Ta có: $$f\left( {a,\sqrt {\frac{{3 - {a^2}}}{2}} ,\sqrt {\frac{{3 - {a^2}}}{2}} } \right) \geqslant 0 \Leftrightarrow 3\left( {{a^3} + 2{{\left( {\sqrt {\frac{{3 - {a^2}}}{2}} } \right)}^3}} \right) + 2a.\frac{{3 - {a^2}}}{2} - 11 \geqslant 0$$$$ \Leftrightarrow 3{a^3} + \frac{{3{{\left( {\sqrt {3 - {a^2}} } \right)}^3}}}{{\sqrt 2 }} + a\left( {3 - {a^2}} \right) - 11 \geqslant 0 \Leftrightarrow \frac{{3{{\left( {\sqrt {3 - {a^2}} } \right)}^3}}}{{\sqrt 2 }} + 9a - 15 + \left( {2a + 4} \right){\left( {a - 1} \right)^2} \geqslant 0$$$$ \Leftrightarrow {\left( {a - 1} \right)^2}\left[ {2a + 4 - \frac{{3\left( {{a^4} + 2{a^3} - 6{a^2} - 14a + 23} \right)}}{{\sqrt 2 \left( {{{\left( {\sqrt {3 - {a^2}} } \right)}^3} + \left( {5 - 3a} \right)\sqrt 2 } \right)}}} \right] \geqslant 0\left( * \right)$$Đặt: $$g\left( a \right) = \left( {2a + 4} \right)\left( {\sqrt 2 {{\left( {\sqrt {3 - {a^2}} } \right)}^3} + 2\left( {5 - 3a} \right)} \right) - 3\left( {{a^4} + 2{a^3} - 6{a^2} - 14a + 23} \right)$$$$ = \left( { - 2{a^3} - 4{a^2} + 6a + 12} \right)\sqrt {6 - 2{a^2}}  - 3{a^4} - 6{a^3} + 6{a^2} + 38a - 29$$Ta có bổ đề sau: $$\sqrt {6 - 2{a^2}}  > \frac{{61}}{{25}} - \frac{{22}}{{25}}a \Leftrightarrow 6 - 2{a^2} > {\left( {\frac{{61}}{{25}} - \frac{{22}}{{25}}a} \right)^2} \Leftrightarrow  - \frac{{1734}}{{625}}{a^2} + \frac{{2684}}{{625}}a + \frac{{29}}{{625}} > 0$$Dễ thấy nó đúng với $a \in \left[ {0;1} \right]$. Mặt khác $ - 2{a^3} - 4{a^2} + 6a + 12 = 2a\left( {1 - a} \right)\left( {a + 3} \right) + 12 > 0\forall a \in \left[ {0;1} \right]$, nên ta được: $$g\left( a \right) > \left( { - 2{a^3} - 4{a^2} + 6a + 12} \right)\left( {\frac{{61}}{{25}} - \frac{{22}}{{25}}a} \right) - 3{a^4} - 6{a^3} + 6{a^2} + 38a - 29$$$$\,\,\,\,\,\,\,\,\,\, = \frac{{ - 31{a^4} - 184{a^3} - 226{a^2} + 1052a + 7}}{{25}} = \frac{{ - 31{a^4} - 184{a^3} + 215{a^2} - 441{a^2} + 1052a + 7}}{{25}}$$Do $ \displaystyle \left\{ \begin{array}{l}-31{{a}^{4}}-184{{a}^{3}}+215{{a}^{2}}>0\forall a\in \left[ {0;1} \right]\\-441{{a}^{2}}+1052a+7>0\forall a\in \left[ {0;1} \right]\\\sqrt{2}\left( {{{{\left( {\sqrt{{3-{{a}^{2}}}}} \right)}}^{3}}+\left( {5-3a} \right)\sqrt{2}} \right)>0\end{array} \right.\Rightarrow g\left( a \right)>0\Rightarrow 2a+4-\frac{{3\left( {{{a}^{4}}+2{{a}^{3}}-6{{a}^{2}}-14a+23} \right)}}{{\sqrt{2}\left( {{{{\left( {\sqrt{{3-{{a}^{2}}}}} \right)}}^{3}}+\left( {5-3a} \right)\sqrt{2}} \right)}}>0$ 

Vậy ta có điều phải chứng minh!




#683879 Đề thi vào 10 chuyên Nguyễn Tất Thành Kon Tum 2017-2018

Đã gửi bởi tuaneee111 on 10-06-2017 - 08:59 trong Tài liệu - Đề thi

Lạm dụng máy tính

ko có máy tính gì ở đây nhé! Đơn giản là sử dụng đẳng thức sau: ${\left( {a + b + c} \right)^2} - 3\left( {ab + bc + ca} \right) = \frac{1}{2}\sum\limits_{cyc} {{{\left( {a - b} \right)}^2}} $, do $a + b + c = 1 \Rightarrow {\left( {a + b + c} \right)^2} = 1$ biến đổi P kiểu này: $$P = 6\left( {ab + bc + ca - \frac{{{{\left( {a + b + c} \right)}^2}}}{3}} \right) + \sum\limits_{cyc} {a{{\left( {a - b} \right)}^2}}  + 2$$$$ =  - \sum\limits_{cyc} {{{\left( {a - b} \right)}^2}}  + \sum\limits_{cyc} {a{{\left( {a - b} \right)}^2}}  + 2 = \sum\limits_{cyc} {{{\left( {a - b} \right)}^2}\left( {a - 1} \right)}  + 2 \leqslant 2$$Được chứ bạn, ở đây ko có lạm dụng gì hết nha!




#683855 Đề thi vào 10 chuyên Nguyễn Tất Thành Kon Tum 2017-2018

Đã gửi bởi tuaneee111 on 09-06-2017 - 21:48 trong Tài liệu - Đề thi

Bạn ghi rõ ra 1 tý được không :v cái đoạn $\sum\limits_{{cyc}}{{{{{\left( {a-b} \right)}}^{2}}\left( {a-1} \right)}}+2\le 2$
À có phải là $(a-b)^2 \geq 0$ mà $a \leq 1$ nên cái đó $ \leq 0$ hả ?

đúng rồi đó bạn :icon6:




#683845 $abc+\frac{17}{ab+bc+ac}\geq 5$

Đã gửi bởi tuaneee111 on 09-06-2017 - 20:49 trong Bất đẳng thức và cực trị

Câu 2.

+ Nếu $ \displaystyle ab+bc+ca\le \frac{9}{4}$ thì bất đẳng thức hiển nhiên đúng.

+ Nếu $ \displaystyle \frac{9}{4}\le ab+bc+ca\le 3$ thì theo $Schur$ ta có: $$ \displaystyle abc\ge \frac{{\left( {a+b+c} \right)\left( {4\left( {ab+bc+ca} \right)-{{{\left( {a+b+c} \right)}}^{2}}} \right)}}{9}=\frac{{4\left( {ab+bc+ca} \right)-9}}{3}$$Khi đó ta được:$$ \displaystyle abc+\frac{{17}}{{ab+bc+ca}}-5\ge \frac{{4\left( {ab+bc+ca} \right)-9}}{3}+\frac{{17}}{{ab+bc+ca}}-5$$$$ = \frac{{4\left( {{{\left( {ab + bc + ca} \right)}^2} - 24\left( {ab + bc + ca} \right) + 51} \right)}}{{3\left( {ab + bc + ca} \right)}} > 0$$Vậy dấu "=" không xảy ra

P/s: Nếu sửa $17$ thành $12$ thì dấu $"="$ sẽ xảy ra




#683841 Đề thi vào 10 chuyên Nguyễn Tất Thành Kon Tum 2017-2018

Đã gửi bởi tuaneee111 on 09-06-2017 - 20:25 trong Tài liệu - Đề thi

Chém bất.

Ta có: $$ \displaystyle P=6\left( {ab+bc+ca} \right)+a{{\left( {a-b} \right)}^{2}}+b{{\left( {b-c} \right)}^{2}}+c{{\left( {c-a} \right)}^{2}}=\sum\limits_{{cyc}}{{{{{\left( {a-b} \right)}}^{2}}\left( {a-1} \right)}}+2\le 2$$Vậy $ \displaystyle \max P=2$. Đẳng thức đặt được khi $ \displaystyle a=b=c=\frac{1}{3}$




#683827 $\frac{b}{{{a^2}}} + \frac{c}{{{b^2}}} + \frac{a}{{{c^2}}...

Đã gửi bởi tuaneee111 on 09-06-2017 - 18:38 trong Bất đẳng thức và cực trị

Bài 2.

Không mất tính tổng quát ta giả sử: $a \geqslant b \geqslant c$

Ta có: $$\frac{{{{\left( {a + b} \right)}^2}}}{{{c^2} + ab}} + \frac{{{{\left( {b + c} \right)}^2}}}{{{a^2} + bc}} + \frac{{{{\left( {c + a} \right)}^2}}}{{{b^2} + ac}} - 6 = \sum\limits_{cyc} {{{\left( {a - c} \right)}^2}\frac{{\left( {a + c} \right)\left( {a + c - b} \right)}}{{\left( {{c^2} + ab} \right)\left( {{a^2} + bc} \right)}}} $$Đăt: $${{S_a} = \frac{{\left( {b + c} \right)\left( {b + c - a} \right)}}{{\left( {{b^2} + ac} \right)\left( {{c^2} + ab} \right)}};\,\,{S_b} = \frac{{\left( {a + c} \right)\left( {a + c - b} \right)}}{{\left( {{c^2} + ab} \right)\left( {{a^2} + bc} \right)}};\,\,{S_c} = \frac{{\left( {a + b} \right)\left( {a + b - c} \right)}}{{\left( {{a^2} + bc} \right)\left( {{b^2} + ca} \right)}}}$$Do $a \geqslant b \geqslant c \Rightarrow {S_b} > 0;\,\,{S_c} > 0$. Bất đẳng thức đúng nếu ta chứng minh được: ${b^2}{S_a} + {a^2}{S_b} \geqslant 0$. Ta có: $${b^2}{S_a} + {a^2}{S_b} = \frac{{{b^2}\left( {b + c} \right)\left( {b + c - a} \right)}}{{{b^2} + ac}} + \frac{{{a^2}\left( {a + c} \right)\left( {a + c - b} \right)}}{{{a^2} + bc}}$$$$ \geqslant \left( {b + c} \right)\left[ {\frac{{{b^2}\left( {b + c - a} \right)}}{{{b^2} + ac}} + \frac{{{a^2}\left( {a + c - b} \right)}}{{{a^2} + bc}}} \right]$$$$ = \frac{{\left( {b + c} \right)\left( {{a^2}{b^2}c + {a^4}c + {a^3}{c^2} - {a^3}bc + {a^2}{b^2}c + {b^4}c - {b^3}ac} \right)}}{{\left( {{b^2} + ac} \right)\left( {{a^2} + bc} \right)}}$$Theo $AM-GM$ ta có: $ \displaystyle \left\{ \begin{array}{l}{{a}^{2}}{{b}^{2}}c+{{a}^{4}}c\ge 2{{a}^{3}}bc>{{a}^{3}}bc\\{{a}^{2}}{{b}^{2}}c+{{b}^{4}}c\ge 2{{b}^{3}}ac>{{b}^{3}}ac\end{array} \right.\Rightarrow {{b}^{2}}{{S}_{a}}+{{a}^{2}}{{S}_{b}}>0$

Vậy có đpcm!




#683824 $\frac{b}{{{a^2}}} + \frac{c}{{{b^2}}} + \frac{a}{{{c^2}}...

Đã gửi bởi tuaneee111 on 09-06-2017 - 18:30 trong Bất đẳng thức và cực trị

Bài 1:

Đặt: $\left( {\frac{1}{a};\frac{1}{b};\frac{1}{c}} \right) \to \left( {x;y;z} \right) \Rightarrow x + y + z = 1 \Rightarrow {\left( {x + y + z} \right)^2} = 1$

Bất đẳng thức trở thành: $$\frac{{{x^2}}}{y} + \frac{{{y^2}}}{z} + \frac{{{z^2}}}{x} \geqslant 3\left( {{x^2} + {y^2} + {z^2}} \right)$$Sử dụng giả thiết ta có: $$\frac{{{x^2}}}{y} + \frac{{{y^2}}}{z} + \frac{{{z^2}}}{x} - 3\left( {{x^2} + {y^2} + {z^2}} \right) = \sum\limits_{cyc} {{{\left( {x - y} \right)}^2}\left( {\frac{1}{y} - 1} \right)}  = \sum\limits_{cyc} {{{\left( {x - y} \right)}^2}\left( {\frac{{x + z}}{y}} \right) \geqslant 0 \Rightarrow Q.E.D} $$




#683797 Đề thi tuyển sinh vào lớp 10 chuyên tỉnh Đắk Lắk

Đã gửi bởi tuaneee111 on 09-06-2017 - 15:19 trong Tài liệu - Đề thi

Ở đây nhé: https://diendantoanh...-lak-2017-2018/




#683796 Chứng minh bất đẳng thức sau lớn hơn 3/4

Đã gửi bởi tuaneee111 on 09-06-2017 - 15:15 trong Bất đẳng thức và cực trị

Đặt: $\left( {\frac{b}{a};\frac{c}{b};\frac{a}{c}} \right) \to \left( {x,y,z} \right) \Rightarrow xyz = 1$. Bất đẳng thức cần chứng minh tương đương với: $$\frac{1}{{{{\left( {x + 1} \right)}^2}}} + \frac{1}{{{{\left( {y + 1} \right)}^2}}} + \frac{1}{{{{\left( {z + 1} \right)}^2}}} \geqslant \frac{3}{4}$$Sử dụng bổ đề quen thuộc: $$\frac{1}{{{{\left( {x + 1} \right)}^2}}} + \frac{1}{{{{\left( {y + 1} \right)}^2}}} \geqslant \frac{1}{{xy + 1}} \Leftrightarrow \frac{{xy{{\left( {x - y} \right)}^2} + {{\left( {xy - 1} \right)}^2}}}{{\left( {xy + 1} \right){{\left( {x + 1} \right)}^2}{{\left( {y + 1} \right)}^2}}} \geqslant 0$$Ta cần chứng minh: $$\frac{1}{{xy + 1}} + \frac{1}{{{{\left( {z + 1} \right)}^2}}} \geqslant \frac{3}{4} \Leftrightarrow \frac{z}{{z + 1}} + \frac{1}{{{{\left( {z + 1} \right)}^2}}} - \frac{3}{4} \geqslant 0 \Leftrightarrow \frac{{{{\left( {z - 1} \right)}^2}}}{{4{{\left( {z + 1} \right)}^2}}} \geqslant 0 \Rightarrow Q.E.D$$P/s: Bài này giống trong đề tuyển sinh chuyên Phan Bội Châu - Nghệ An




#683794 Đề tuyển sinh vào 10 THPT chuyên Phan Bội Châu - Nghệ An 2017-2018

Đã gửi bởi tuaneee111 on 09-06-2017 - 15:09 trong Tài liệu - Đề thi

Chém bất.

Đặt $ \displaystyle \left( {\frac{b}{a};\frac{c}{b};\frac{a}{c}} \right)\to \left( {x,y,z} \right)\left( {z\le 1} \right)$$ \displaystyle \Rightarrow xyz=1$. Bất đẳng thức trở thành: $$ \displaystyle \frac{1}{{{{{\left( {x+1} \right)}}^{2}}}}+\frac{1}{{{{{\left( {y+1} \right)}}^{2}}}}+\frac{4}{{{{{\left( {z+1} \right)}}^{2}}}}\ge \frac{3}{2}$$Ta có bổ đề quen thuộc sau: $$ \displaystyle \frac{1}{{{{{\left( {x+1} \right)}}^{2}}}}+\frac{1}{{{{{\left( {y+1} \right)}}^{2}}}}\ge \frac{1}{{xy+1}}\Leftrightarrow \frac{{xy{{{\left( {x-y} \right)}}^{2}}+{{{\left( {xy-1} \right)}}^{2}}}}{{\left( {xy+1} \right){{{\left( {x+1} \right)}}^{2}}{{{\left( {y+1} \right)}}^{2}}}}\ge 0$$Áp dụng vào bài toán ta cần chứng minh: $$ \displaystyle \frac{1}{{xy+1}}+\frac{4}{{{{{\left( {z+1} \right)}}^{2}}}}\ge \frac{3}{2}\Leftrightarrow \frac{z}{{z+1}}+\frac{4}{{{{{\left( {z+1} \right)}}^{2}}}}-\frac{3}{2}\ge 0\Leftrightarrow \frac{{\left( {1-z} \right)\left( {z+5} \right)}}{{2{{{\left( {z+1} \right)}}^{2}}}}\ge 0\Rightarrow Q.E.D$$Bất đẳng thức cuối đúng nên có đpcm




#683609 $P=\sqrt{a+\frac{(b-c)^2}{4}}+\sqrt{b+\frac{(a-c)^2}...

Đã gửi bởi tuaneee111 on 08-06-2017 - 07:16 trong Bất đẳng thức và cực trị

Sử dụng giả thiết ta có: $$P = \sum\limits_{cyc} {\sqrt {a + \frac{{{{\left( {b - c} \right)}^2}}}{4}} }  = \sum\limits_{cyc} {\sqrt {a\left( {a + b + c} \right) + {{\left( {\frac{{b + c}}{2}} \right)}^2} - bc}  = \sum\limits_{cyc} {\sqrt {{{\left( {a + \frac{{b + c}}{2}} \right)}^2} - bc} } } $$$$ \leqslant \sum\limits_{cyc} {\left( {a + \frac{{b + c}}{2}} \right)}  = 2\left( {a + b + c} \right) = 2$$




#683565 Đề thi vào 10 chuyên tỉnh Đak Lak 2017-2018

Đã gửi bởi tuaneee111 on 07-06-2017 - 20:01 trong Tài liệu - Đề thi

Chém bất.

Theo $Cauchy - Schwarz$ dạng $Engel$ ta có: $$ \displaystyle \frac{a}{{b+c}}+\frac{b}{{c+a}}+\frac{{4c}}{{a+b}}\ge \frac{{{{{\left( {a+b+2c} \right)}}^{2}}}}{{2\left( {ab+bc+ca} \right)}}$$Mặt khác ta lại có: $ \displaystyle {{\left( {a+b+2c} \right)}^{2}}-4\left( {ab+bc+ca} \right)={{\left( {a-b} \right)}^{2}}+4{{c}^{2}}>0$

Vậy bất đẳng thức được chứng minh thành công!




#683562 Đề thi vào 10 chuyên tỉnh Hà Tĩnh 2017-2018

Đã gửi bởi tuaneee111 on 07-06-2017 - 19:54 trong Tài liệu - Đề thi

Câu phương trình: $$ \displaystyle \begin{array}{l}4{{x}^{2}}=\left( {3x-2} \right){{\left( {\sqrt{{2x+1}}-1} \right)}^{2}}\\\Leftrightarrow {{\left( {\sqrt{{2x+1}}-1} \right)}^{2}}{{\left( {\sqrt{{2x+1}}+1} \right)}^{2}}=\left( {3x-2} \right){{\left( {\sqrt{{2x+1}}-1} \right)}^{2}}\\\Leftrightarrow {{\left( {\sqrt{{2x+1}}-1} \right)}^{2}}\left( {{{{\left( {\sqrt{{2x+1}}+1} \right)}}^{2}}-\left( {3x-2} \right)} \right)=0\\\Leftrightarrow {{\left( {\sqrt{{2x+1}}-1} \right)}^{2}}\left( {{{{\left( {\sqrt{{2x+1}}+1} \right)}}^{2}}-\frac{3}{2}{{{\left( {\sqrt{{2x+1}}} \right)}}^{2}}+\frac{7}{2}} \right)=0\\\Leftrightarrow -\frac{1}{2}{{\left( {\sqrt{{2x+1}}-1} \right)}^{2}}\left( {\sqrt{{2x+1}}-2+\sqrt{{13}}} \right)\left( {\sqrt{{2x+1}}-2-\sqrt{{13}}} \right)=0\\\Leftrightarrow \left[ \begin{array}{l}x=0\\\sqrt{{2x+1}}=2+\sqrt{{13}}\Leftrightarrow x=8+2\sqrt{{13}}\end{array} \right.\end{array}$$




#683555 Đề thi vào 10 chuyên tỉnh Đồng Nai 2017-2018

Đã gửi bởi tuaneee111 on 07-06-2017 - 19:38 trong Tài liệu - Đề thi

Chém câu bất.

Không mất tính tổng quát ta giả sử $c = \min \left\{ {a,b,c} \right\}$.

Ta có: $$ \displaystyle \left( {\sum\limits_{{cyc}}{{ab}}} \right)\left( {\sum\limits_{{cyc}}{{{{a}^{2}}}}} \right)-\sum\limits_{{cyc}}{{ab\left( {{{b}^{2}}+bc+ca} \right)}}=c\left( {a+b} \right){{\left( {a-b} \right)}^{2}}+b\left( {b+c} \right)\left( {a-c} \right)\left( {b-c} \right)\ge 0$$




#683538 Đề thi chuyên toán tỉnh Thái Bình 2017 - 2018

Đã gửi bởi tuaneee111 on 07-06-2017 - 17:50 trong Tài liệu - Đề thi

 

Bài bất chắc cách này chắc đơn giản nhất r.
$\frac{1}{{a\sqrt {3a + 2b} }} + \frac{1}{{b\sqrt {3b + 2c} }} + \frac{1}{{c\sqrt {3c + 2a} }} \ge \frac{3}{{\sqrt {5abc} }}\\ \frac{1}{{a\sqrt {3a + 2b} }} + \frac{1}{{b\sqrt {3b + 2c} }} + \frac{1}{{c\sqrt {3c + 2a} }} \ge \frac{3}{{\sqrt[3]{{abc\sqrt {(3a + 2b)(3b + 2c)(3c + 2a)} }}}}\\ CM:\sqrt {5abc} \le \sqrt[3]{{abc\sqrt {(3a + 2b)(3b + 2c)(3c + 2a)} }}\\ \Leftrightarrow 125{a^3}{b^3}{c^3} \le {a^2}{b^2}{c^2}(3a + 2b)(3b + 2c)(3c + 2a)\\ \Leftrightarrow (3a + 2b)(3b + 2c)(3c + 2a) \ge 125abc\\ \Leftrightarrow 27abc + 18{b^2}c + 18a{c^2} + 12b{c^2} + 18{a^2}b + 12a{b^2} + 12{a^2}c + 8abc \ge 125abc\\ \Leftrightarrow 18({a^2}b + {b^2}c + {c^2}a) + 12(a{b^2} + b{c^2} + c{a^2}) \ge 90abc\\ \Leftrightarrow 3({a^2}b + {b^2}c + {c^2}a) + 2(a{b^2} + b{c^2} + c{a^2}) \ge 15abc$

 

hình như lời giải có vấn đề, đáng lẽ phải chứng minh $\sqrt {5abc}  \geqslant \root 3 \of {abc\sqrt {\left( {3a + 2b} \right)\left( {3b + 2c} \right)\left( {3c + 2a} \right)} } $, mà điều này thì lại ko đúng!




#683536 $\boxed{\text{Chuyên Đề}}$ Bất đẳng thức - Cực trị

Đã gửi bởi tuaneee111 on 07-06-2017 - 17:28 trong Bất đẳng thức và cực trị

mọi người giải hộ cho mình bài này với

chứng minh BĐT a/b+b/c+c/a>=b/a+a/c+c/b ( với a>=b>=c>0)

bài này bạn có đăng ở đây và mk cx có trả lời rồi mà: https://diendantoanh...-bất-đẳng-thức/




#683519 Tìm GTNN của P=$\frac{x}{\sqrt{1-x}...

Đã gửi bởi tuaneee111 on 07-06-2017 - 14:25 trong Đại số

Anh dùng theo Cauchy-Schwars đi như chị trên nói ấy, vì BĐT thực ra e vẫn mù

Theo $Cauchy - Schwarz$ ta có: $$a + b + c \leqslant \sqrt {3\left( {{a^2} + {b^2} + {c^2}} \right)} $$$$\left( {{a^3} + {b^3} + {c^3}} \right)\left( {a + b + c} \right) = \left( {{{\left( {\sqrt {{a^3}} } \right)}^2} + {{\left( {\sqrt {{b^3}} } \right)}^2} + {{\left( {\sqrt {{c^3}} } \right)}^2}} \right)\left( {{{\left( {\sqrt a } \right)}^2} + {{\left( {\sqrt b } \right)}^2} + {{\left( {\sqrt c } \right)}^2}} \right) \geqslant {\left( {{a^2} + {b^2} + {c^2}} \right)^2}$$$$ \Rightarrow {a^3} + {b^3} + {c^3} \geqslant \frac{{{{\left( {{a^2} + {b^2} + {c^2}} \right)}^2}}}{{a + b + c}} \geqslant \frac{{{{\left( {{a^2} + {b^2} + {c^2}} \right)}^2}}}{{\sqrt {3\left( {{a^2} + {b^2} + {c^2}} \right)} }} = 81$$ :D  :icon6:  :ukliam2:




#683504 Nguyên lý Dirichlet

Đã gửi bởi tuaneee111 on 07-06-2017 - 12:02 trong Số học

Ôi má ơi toàn tiếng Lào, campuchia gì vậy! :D  :(  :icon6:  chả hiểu gì




#683502 $\left (a^{2}+1 \right )\left (b^{2}...

Đã gửi bởi tuaneee111 on 07-06-2017 - 11:48 trong Bất đẳng thức và cực trị

ở đây nha: https://diendantoanh...ng/#entry683001




#683476 Tìm GTNN : P = $\frac{1}{6a+1} + \frac...

Đã gửi bởi tuaneee111 on 07-06-2017 - 08:59 trong Bất đẳng thức và cực trị

Ta có: $$\frac{1}{{6x + 1}} - \frac{{27}}{{49\left( {2x + 1} \right)}} + \frac{2}{{49}} = \frac{{24{{\left( {x - 1} \right)}^2}}}{{49\left( {6x + 1} \right)\left( {2x + 1} \right)}} \geqslant 0$$Do đó ta được: $$P = \frac{1}{{6a + 1}} + \frac{1}{{6b + 1}} + \frac{1}{{6c + 1}} \geqslant \frac{{27}}{{49}}\left( {\frac{1}{{2a + 1}} + \frac{1}{{2b + 1}} + \frac{1}{{2c + 1}}} \right) - \frac{6}{{49}} \geqslant \frac{3}{7}$$




#683475 GTLN $$P = {\left( {x + y + z} \right)^2} - \frac{{...

Đã gửi bởi tuaneee111 on 07-06-2017 - 08:54 trong Bất đẳng thức và cực trị

Thế giả thiết vào P ta được: $$P = {\left( {x + y + z} \right)^2} - \frac{{{x^3} + {y^3} + {z^3}}}{{9xyz}} + \frac{3}{{xy + yz + xz}} =  - \sum\limits_{cyc} {{{\left( {x - y} \right)}^2}\left( {\frac{{\sum\limits_{cyc} {z{{\left( {x - y} \right)}^2}} }}{{18xyz}} + 1} \right)}  + \frac{{29}}{3} \leqslant \frac{{29}}{3}$$Vậy $\max P = \frac{{29}}{3}$




#683471 Tìm GTNN của P=$\frac{x}{\sqrt{1-x}...

Đã gửi bởi tuaneee111 on 07-06-2017 - 08:29 trong Đại số

Ta có: $$\sum\limits_{cyc} {{x^2}.\underbrace {1.1...1.1}_{1007\,\,Numbers}} \mathop  \leqslant \limits^{AM - GM} \sum\limits_{cyc} {\frac{{{x^{2016}} + 1007}}{{1008}}}  = \frac{{3 + 3.1007}}{{1008}} = 3$$Vậy $\max P = 3$




#683469 Tìm GTNN của P=$\frac{x}{\sqrt{1-x}...

Đã gửi bởi tuaneee111 on 07-06-2017 - 08:21 trong Đại số

Câu 2:

Biến đổi giả thiết và sử dụng $AM-GM$ ta có: $$P = \frac{x}{{\sqrt {1 - x} }} + \frac{y}{{\sqrt {1 - y} }} = \frac{x}{{\sqrt y }} + \frac{y}{{\sqrt x }} = \frac{{\left( {\sqrt x  + \sqrt y } \right)\left( {x + y - \sqrt {xy} } \right)}}{{\sqrt {xy} }}$$$$ \geqslant \frac{{2\root 4 \of {xy} \left( {x + y - \sqrt {xy} } \right)}}{{\sqrt {xy} }} = \frac{{2\left( {x + y - \sqrt {xy} } \right)}}{{\root 4 \of {xy} }} \geqslant \frac{{2\left( {x + y - \frac{{x + y}}{2}} \right)}}{{\sqrt {\frac{{x + y}}{2}} }} = \sqrt 2 $$Vậy $\min P = \sqrt 2 $




#683467 Tìm GTNN của P=$\frac{x}{\sqrt{1-x}...

Đã gửi bởi tuaneee111 on 07-06-2017 - 08:07 trong Đại số

Câu 1:

Theo $Holder$ ta có: $$\left( {1 + 1 + 1} \right)\left( {{a^3} + {b^3} + {c^3}} \right)\left( {{a^3} + {b^3} + {c^3}} \right) \geqslant {\left( {{a^2} + {b^2} + {c^2}} \right)^3}$$$$ \Rightarrow {a^3} + {b^3} + {c^3} \geqslant \sqrt {\frac{{{{\left( {{a^2} + {b^2} + {c^2}} \right)}^3}}}{3}}  = 81 \Rightarrow Q.E.D$$P/s: Hình như đề bài phải là chứng minh ${a^2} + {b^3} + {c^3} \geqslant 81$